Solution to Incorrect Homework Equation

  • Thread starter Thread starter chapsticks
  • Start date Start date
  • Tags Tags
    Homework
Click For Summary
SUMMARY

The discussion centers on solving the integral for the arc length of a function defined by the equation s=∫ab√(1+[f'(x)]²)dx. The user initially misapplied the limits of integration, using [0, 1] instead of the correct interval [0, 6]. The correct antiderivative is presented as (2/3)(1 + x)^(3/2) evaluated from 0 to 6, confirming the solution's accuracy. The use of LaTeX for formatting the integral and antiderivative is also highlighted.

PREREQUISITES
  • Understanding of integral calculus, specifically arc length calculations.
  • Familiarity with LaTeX for mathematical notation.
  • Knowledge of derivatives and their application in integrals.
  • Ability to interpret and manipulate inequalities in calculus.
NEXT STEPS
  • Study the properties of arc length integrals in calculus.
  • Learn how to format mathematical expressions using LaTeX.
  • Explore the concept of limits of integration in definite integrals.
  • Review the relationship between derivatives and integrals in calculus.
USEFUL FOR

Students studying calculus, educators teaching integral calculus, and anyone looking to improve their skills in solving definite integrals and using LaTeX for mathematical expressions.

chapsticks
Messages
38
Reaction score
0

Homework Statement



so I did

Homework Equations



s=∫ba√(1+[f'(x)]2dx

The Attempt at a Solution


y=2/3x3/2+7
y'=x1/2 0≤x≤1
=∫6√(1+(x1/2)2)dx
=[2/3(1+x)3/2]7

my answer came out wrong
 

Attachments

  • 29.jpg
    29.jpg
    10.3 KB · Views: 442
Physics news on Phys.org
chapsticks said:

Homework Statement



so I did

Homework Equations



s=∫ba√(1+[f'(x)]2dx

The Attempt at a Solution


y=2/3x3/2+7
y'=x1/2 0≤x≤1
Why do you have this inequality?
chapsticks said:
=∫6√(1+(x1/2)2)dx
=[2/3(1+x)3/2]7

my answer came out wrong
What is that 7 doing?
 
I saw it in an example for the inequality.. The 7 is for the a & b part I don't know how to place the 0 on the bottom.
 
chapsticks said:
I saw it in an example for the inequality.. The 7 is for the a & b part I don't know how to place the 0 on the bottom.
Your inequality is saying that the interval is [0, 1]. It's actually [0, 6].

Here's how your integral looks using LaTeX.
\int_0^6 \sqrt{1+x}~dx

Here's how to write that integral in LaTeX.

[ tex]\int_0^6 \sqrt{1+x}~dx[ /tex]

(The extra spaces in the tex and /tex tags prevent the browser from rendering the code inside.)

Here's the antiderivative with limits of integration shown.
\left. \frac{2}{3}(1 + x)^{3/2}\right|_0^6

The LaTeX for that.
[ tex]\left. \frac{2}{3}(1 + x)^{3/2}\right|_0^6[ /tex]
 
I got it correct :))
 

Attachments

  • 29.jpg
    29.jpg
    11.1 KB · Views: 476
Good to know. That's what I got, too.
 
Question: A clock's minute hand has length 4 and its hour hand has length 3. What is the distance between the tips at the moment when it is increasing most rapidly?(Putnam Exam Question) Answer: Making assumption that both the hands moves at constant angular velocities, the answer is ## \sqrt{7} .## But don't you think this assumption is somewhat doubtful and wrong?

Similar threads

  • · Replies 2 ·
Replies
2
Views
2K
  • · Replies 10 ·
Replies
10
Views
2K
  • · Replies 6 ·
Replies
6
Views
2K
  • · Replies 10 ·
Replies
10
Views
2K
  • · Replies 4 ·
Replies
4
Views
2K
Replies
4
Views
2K
  • · Replies 7 ·
Replies
7
Views
2K
  • · Replies 2 ·
Replies
2
Views
1K
  • · Replies 11 ·
Replies
11
Views
3K
  • · Replies 5 ·
Replies
5
Views
1K